jgmartin82
Thanks Received: 15
Vinny Gambini
Vinny Gambini
 
Posts: 18
Joined: November 01st, 2011
 
 
 

Q12 - Journalist: A book claiming that a new drug

by jgmartin82 Mon Apr 29, 2013 4:44 pm

PT68, S2, Q12 (Identify a Flaw)

(D) is correct.


We’re asked to find a flaw in the journalist’s reasoning, so let’s figure out the journalist’s argument. We get some background about a physician disagreeing with a book that is claiming a drug is dangerous. The next line seems to give us an argument with the X, hence Y language. It looks like this:

Phys. employed by drug maker --> Phys. probably has personal reasons to critique the book

But then we get a big fat therefore introducing our main conclusion: the physician’s critique of the book isn’t legitimate. We end up with a P-->IC-->C structure.

It’s vital to have a sense of the flaw before going into the answer choices, so let’s look for one, focusing on the second gap, as that’s typically where the LSAT will attack. Indeed, there’s a glaring flaw here. Regardless of the physician’s motives, she still may have a great argument. If your friend invited you to the waterpark, claiming that you’ll have a great time on the waterslides, would you change you decide not to go if you found out that your friend was benefitting from a discount ticket deal because you went? No!

One other thing we may have noted here is that there is a lot of critiquing going on. We are meant to critique a journalist who critiqued a physician who critiqued a book that critiqued a drug for its side effects. Let’s make sure we don’t get turned around on any of that in the answer choices.

(A) has no bearing on the conclusion because our conclusion only deals with the physician’s attack on dangerous side effects. Other claims in the book are irrelevant. Eliminate.

(B) addresses the flaw we brought up, let’s keep it.

(C) has no bearing on the conclusion. We don’t care about the author of the book, we only care about the physician’s critique of the book! Eliminate.

(D) this also addresses the flaw we brought up; keep it.

(E) is tempting, but it’s not a flaw in the argument; it’s actually a flaw in the inverse of our argument. We want something stating that even if someone has personal bias, her claims aren’t necessarily illegitimate. Instead, we get even if someone’s claims are illegitimate, they don’t necessarily have personal bias. Notice this answer starts with the idea that a critique is questionable, while we want to end there! We’ve reversed the logic. Eliminate.

Down to (B) and (D). In the end, (B) has a couple of problems. First, it doesn’t address a gap, it jumps from the premise about employment all the way to the conclusion about ability to critique fairly. Even aside from that, the language anyone remotely associated with a company should raise eyebrows and in fact does (B) in as well. The issue at hand is the difference between necessary and sufficient assumptions. The language takes for granted should introduce a necessary assumption. Did our argument require that anyone remotely associated with the company be biased? It certainly would help the argument, but no, it’s not necessary.

(D) on the other hand introduces exactly the flaw we predicted by presenting an alternative possibility, namely that one can have personal connections to an issue while still providing legitimate critiques of it.
 
karin.yoo
Thanks Received: 0
Forum Guests
 
Posts: 3
Joined: September 02nd, 2012
 
 
 

Re: Q12 - Journalist: A book claiming that a new drug

by karin.yoo Thu Jun 06, 2013 1:41 pm

Nice explanation above.
What threw me off in B is the phrase "unable to".
The author is merely saying that the physician is DENYING the side effects -- this isn't the same as the physician being NOT ABLE to realize the side effects.
 
ilia.medovikov
Thanks Received: 5
Forum Guests
 
Posts: 13
Joined: July 02nd, 2013
 
 
 

Re: Q12 - Journalist: A book claiming that a new drug

by ilia.medovikov Sat Jul 27, 2013 5:31 pm

Hello,

I was wondering if someone could elaborate more on why (e) is incorrect. I agree with jgmartin82's logic: "We want something stating that even if someone has personal bias, her claims aren’t necessarily illegitimate." In an diagrammed form, this statement looks as follows: PB -----> ~I.
PB= personal bias
I= illegitimate

However, when I look at (e) it seems to read as: I----->~PB. The contrapositive is PB------>~I, which appears to describe the flaw.

Thank you in advance for the help!
 
kjsmit02
Thanks Received: 2
Vinny Gambini
Vinny Gambini
 
Posts: 16
Joined: January 07th, 2015
 
 
 

Re: Q12 - Journalist: A book claiming that a new drug

by kjsmit02 Sat Aug 29, 2015 12:55 pm

karin.yoo Wrote:Nice explanation above.
What threw me off in B is the phrase "unable to".
The author is merely saying that the physician is DENYING the side effects -- this isn't the same as the physician being NOT ABLE to realize the side effects.


(B) Also slipped me up with "unable", but I looked at this a little different and am unsure if my reasoning is correct. Essentially, (B) tells us that the argument failed to consider that anyone involved with the company would be unable to present an fair critique. But isn't this exactly what the argument succeeded in doing? I think that the use of the double negative in "takes for granted" and "unable" confuses things. If (B) had instead replaced "unable" with "able" and fixed the strength issue of "remotely associated" with simply "associated", then answer choice (B) would be correct, right? I had failed to even read (D), which now seems to mirror this corrected version of (B).
 
zdlsat
Thanks Received: 0
Vinny Gambini
Vinny Gambini
 
Posts: 13
Joined: July 23rd, 2014
 
 
 

Re: Q12 - Journalist: A book claiming that a new drug

by zdlsat Sun Oct 04, 2015 5:26 pm

kjsmit02 Wrote:
karin.yoo Wrote:Nice explanation above.
What threw me off in B is the phrase "unable to".
The author is merely saying that the physician is DENYING the side effects -- this isn't the same as the physician being NOT ABLE to realize the side effects.


(B) Also slipped me up with "unable", but I looked at this a little different and am unsure if my reasoning is correct. Essentially, (B) tells us that the argument failed to consider that anyone involved with the company would be unable to present an fair critique. But isn't this exactly what the argument succeeded in doing? I think that the use of the double negative in "takes for granted" and "unable" confuses things. If (B) had instead replaced "unable" with "able" and fixed the strength issue of "remotely associated" with simply "associated", then answer choice (B) would be correct, right? I had failed to even read (D), which now seems to mirror this corrected version of (B).



I think the problem here is necessary and sufficient assumptions.
Like jgmartin82 mentioned, " it takes for granted that..." should follow by a necessary assumption.
The journalist takes granted that this specific physician could not provide legitimate grounds bc he is employed by that company. It doesn't have to be anyone. All the journalist cares here is this specific physician.
In the matter of deny vs unable to fairly weigh evidences, I don't really think it's a problem. Yes, it is a term shift, but I think fairly weigh evidences is necessary for legitimate deny, i.e. a legitimate deny requires fairly weigh evidences.

~(fairly weigh evidences) ---> ~(legitimate deny)


If this physician can't fairly weigh evidence, how he can make a legitimate deny.

So the author is taking granted that this physician is bias, not anyone associated is bias.
 
TillyS471
Thanks Received: 1
Vinny Gambini
Vinny Gambini
 
Posts: 8
Joined: September 05th, 2018
 
 
 

Re: Q12 - Journalist: A book claiming that a new drug

by TillyS471 Mon Sep 10, 2018 9:59 pm

To me, B feels like a typical LSAT trap, trying to make the answer really sound attractive, misleading someone in a rush to think this is classic Ad hom answer when it's not.
Clues of it being a trap are very slight, and hard to detect: Anyone even remotely associated ? Like the employee's second cousin?